Prep test qn. - Pls explain

This topic has expert replies
Junior | Next Rank: 30 Posts
Posts: 24
Joined: Thu Jan 18, 2007 10:03 pm

Prep test qn. - Pls explain

by pavithra » Sat Mar 24, 2007 2:08 pm
Hi

I got this qn. in MGMAT prep test


Is x > y?

(1) sq.rt (x) > y

(2) x^3 > y

The answer is C. Could someone explain the solution pls? (I don't understand the explanation by MGMAT)

Thanks

Pavithra

User avatar
Legendary Member
Posts: 2469
Joined: Thu Apr 20, 2006 12:09 pm
Location: BtG Underground
Thanked: 85 times
Followed by:14 members

Re: Prep test qn. - Pls explain

by aim-wsc » Tue Mar 27, 2007 12:16 pm
pavithra wrote:Hi

I got this qn. in MGMAT prep test


Is x > y?

(1) sq.rt (x) > y

(2) x^3 > y

The answer is C. Could someone explain the solution pls? (I don't understand the explanation by MGMAT)

Thanks

Pavithra
1) for sq rt (x) >y
is alone not sufficient since sq rt can be of -ve value also:
for example: x=4 & y=1 then
sq rt (x) = +/- 2
Insufficient.

2) for x^3 >y
is alone again makes trouble when we take values of x less than and more than one... eg take x=0.2 or x=2
NO issues with -ve +ve since the sign of the cube of any number remains same as the original number's.

so when you combine both statement you get exact idea of number whether it is +ve or -ve and more than 1 or in fraction.

Senior | Next Rank: 100 Posts
Posts: 30
Joined: Mon Oct 15, 2007 11:47 pm
Thanked: 1 times

by ash g » Sun Mar 09, 2008 10:27 am
All,
sorry, reopening an old question but did not understand the explanation below.

1) for sq rt (x) >y
is alone not sufficient since sq rt can be of -ve value also:
for example: x=4 & y=1 then
sq rt (x) = +/- 2
Insufficient.


So if both +2 and -2 are greater than y implies y less than -2 and less than x.
I must be missing something very simple here...but i think answer should be A.

Thx,
Ash

Senior | Next Rank: 100 Posts
Posts: 55
Joined: Tue Mar 04, 2008 6:41 pm
Thanked: 2 times
Followed by:1 members

by smkrn » Sun Mar 09, 2008 10:52 am
ash g wrote:All,
sorry, reopening an old question but did not understand the explanation below.

1) for sq rt (x) >y
is alone not sufficient since sq rt can be of -ve value also:
for example: x=4 & y=1 then
sq rt (x) = +/- 2
Insufficient.


So if both +2 and -2 are greater than y implies y less than -2 and less than x.
I must be missing something very simple here...but i think answer should be A.

Thx,
Ash
I think the problem with part 1) is that sqrt(x) could be larger or smaller than 1.

For example, if sqrt(x)=0.2 and y=0.1, then sqrt(x) is greater than y but x=.04 is less than y. However if sqrt(x)=2 and y=1, sqrt(x) is greater than y and x=4 is greater than y. So part 1) is not sufficient by itself. Hopefully this makes sense.

Senior | Next Rank: 100 Posts
Posts: 30
Joined: Mon Oct 15, 2007 11:47 pm
Thanked: 1 times

by ash g » Sun Mar 09, 2008 10:59 am
Valid. Many thanks. I just got caught up in the 4 & 1 example that I forgot about using fractions.
Regards,
Ash